LSAT and Law School Admissions Forum

Get expert LSAT preparation and law school admissions advice from PowerScore Test Preparation.

User avatar
 Dave Killoran
PowerScore Staff
  • PowerScore Staff
  • Posts: 5850
  • Joined: Mar 25, 2011
|
#59781
Complete Question Explanation
(The complete setup for this game can be found here: lsat/viewtopic.php?t=1655)

The correct answer choice is (C)

The question stem asserts that only one of the five sites dates from the 10th century, and because we have already established that the third site dates from the 10th century, we know that the third site is the only site from the 10th century. In addition, from the third rule we know that G must discover a site from the 10th century, and so we can conclude that G must discover the third site. In this Cannot Be True question, then, F cannot discover the third site and answer choice (C) is correct.

Note that although other deductions could be made (such as that the fifth site dates from the 9th century), and an entire diagram could be drawn out for this question, there is no point in doing so since our initial diagram already determined that the third site was from the 10th century, and that provides the starting point for a fast and clear path to the correct answer.

Answer choices (A), (B), (D), and (E) are each incorrect because F could discover any of the sites listed in these answer choices.
 joliekwok8@gmail.com
  • Posts: 14
  • Joined: Jun 21, 2023
|
#102408
Hi,

I'm a bit confused as to why only the 3rd site can be from the 10th century? From the initial setup, it looks like it can be F/G and 9/10. Any help would be appreciated!

Thanks!
 Adam Tyson
PowerScore Staff
  • PowerScore Staff
  • Posts: 5153
  • Joined: Apr 14, 2011
|
#102409
The setup for this game is explained in this thread, joliekwok8@gmail.com :

viewtopic.php?f=221&t=1655

There, you'll see that the site visited third must always be from the 10th Century, because the fourth site cannot be from the 8th Century and the site visited third must be more recent than the one visited fourth.

In this local question, where there is only one site from the 10th Century, that therefore has to be the third site, and G has to have discovered it. This question eliminates the possibility of F going third and G going fifth.

Get the most out of your LSAT Prep Plus subscription.

Analyze and track your performance with our Testing and Analytics Package.